Need help evaluating an improper integral as a power series.

Click For Summary
The discussion revolves around evaluating the improper integral of x^2ln(1+x) using power series, with the book providing an answer that includes a series and a radius of convergence of R = 1. The user attempts to derive the same result but encounters discrepancies in the series representation, particularly with the coefficients and signs. Despite following the integration process and referencing established series expansions, the user cannot reconcile their answer with the book's. They express frustration over a potential sign error in the book's solution and suggest differentiating to verify the correctness of their findings. The conversation emphasizes the challenges of matching results in power series evaluations.
uchuu-man chi
Messages
5
Reaction score
0

Homework Statement



Evaluate the indefinite integral as a power series. What is the radius of convergence (R)?
##\int x^2ln(1+x) \, dx##

Book's answer: ##\int x^2ln(1+x) dx = C + \sum_{n=1}^\infty (-1)^n \frac {x^{n+3}} {n(n+3)}; R = 1##

Homework Equations


Geometric series
##\frac {1} {1-x} = \sum_{n=0}^\infty x^n ; |x|<1##

The Attempt at a Solution


##\frac {1} {1-x} = \sum_{n=0}^\infty x^n ; |x|<1##

-Substitute -x in for x
##\frac {1} {1+x} = \sum_{n=0}^\infty (-1)^n x^n ; |x|<1##

-Integrate
##\int \frac {1} {1+x} dx = \int \sum_{n=0}^\infty (-1)^n x^n dx ; |x|<1 \\ = ln(1+x) = C + \sum_{n=0}^\infty (-1)^n \frac {x^{n+1}}{n+1} ; |x|<1##

##\text {When x=0, C = ln(1) = 0} \\ = ln(1+x) = \sum_{n=0}^\infty (-1)^n \frac {x^{n+1}}{n+1} ; |x|<1##

-Multiply by x2
##x^2 ln(1+x) = x^2 \sum_{n=0}^\infty (-1)^n \frac {x^{n+1}} {n+1} ; |x|<1 \\ = x^2ln(1+x) = \sum_{n=0}^\infty (-1)^n \frac {x^{n+3}} {(n+1)}; |x|<1##

-Integrate

##\int x^2 ln(1+x) dx = \int \sum_{n=0}^\infty (-1)^n \frac {x^{n+3}} {n+1} dx ; |x|<1 \\ = \int x^2 ln(1+x) dx = C + \sum_{n=0}^\infty (-1)^n \frac {x^{n+4}} {(n+1)(n+4)} , R = 1##

-My answer
##C + \sum_{n=0}^\infty (-1)^n \frac {x^{n+4}} {(n+1)(n+4)} , R = 1##

I've been trying to get the same answer as the book, but even if I shifted the index to start at 1, it would be ##\sum_{n=1}^\infty (-1)^{n+1} \frac {x^{n+3}} {n(n+3)} \\ \text {or} \\ \sum_{n=1}^\infty (-1)^{n-1} \frac {x^{n+3}} {n(n+3)}##

I can't seem to find where I made a mistake. Can someone please help me? Thank you in advance.
 
Last edited:
Physics news on Phys.org
I can't find a sign error and Wikipedia is also on our side: ##ln(1+x)=-\sum_{n\geq 0}\frac{(-x)^{n+1}}{n+1}## which I took for comparison to what the differentiation of the book's answer got me. So as long as I didn't fall into the same pit as you, the sign error is in the book. You could differentiate, too, just to be sure.
 
  • Like
Likes uchuu-man chi
Question: A clock's minute hand has length 4 and its hour hand has length 3. What is the distance between the tips at the moment when it is increasing most rapidly?(Putnam Exam Question) Answer: Making assumption that both the hands moves at constant angular velocities, the answer is ## \sqrt{7} .## But don't you think this assumption is somewhat doubtful and wrong?

Similar threads

  • · Replies 3 ·
Replies
3
Views
1K
Replies
8
Views
3K
  • · Replies 2 ·
Replies
2
Views
2K
  • · Replies 14 ·
Replies
14
Views
3K
  • · Replies 7 ·
Replies
7
Views
2K
  • · Replies 3 ·
Replies
3
Views
2K
  • · Replies 13 ·
Replies
13
Views
2K
  • · Replies 14 ·
Replies
14
Views
2K
  • · Replies 8 ·
Replies
8
Views
2K
  • · Replies 2 ·
Replies
2
Views
2K